Freitag, 20. Dezember 2013

Übungen zur Analysis I (nicht vertieft): Alternativer alternativer Zugang zu Blatt 8, Aufgabe 4 b)

Als Reaktion auf meinen kürzlich veröffentlichten Eintrag „Übungen zur Analysis I (nicht vertieft): Alternativer Zugang zu Blatt 8, Aufgabe 4 b)“ erhielt ich von Herrn Prof. Rieder Nachricht über ein weiteres Argument, das beweist, dass für alle Folgen reeller Zahlen $(a_n)_{n\in\N}$ und alle $k\in\N_0$ die Beziehung
\begin{equation}
 \limsup_{n\to\infty} \sqrt[n]{|a_n|} = \limsup_{n\to\infty} \sqrt[n]{|a_{n+k}|} \label{e_main}
\end{equation} gilt. Die oberen Limiten werden dabei in \eqref{e_main} sowie im gesamten weiteren Verlauf stets bezogen auf die erweiterten reellen Zahlen $\bar\R = \R\cup\{\pm\infty\}$ berechnet. Ich gebe Herrn Prof. Rieders Argument sinngemäß wieder (und hoffe, er möge mir kleinere Abweichungen von seiner übermittelten Darstellung verzeihen).
Lemma. Sei $n_0\in\N$, $(b_n)_{n\geq n_0}$ eine Folge in $[0,\infty)$, $k\in\Z$. Dann gilt
\begin{equation}
 l := \limsup_{\substack{n\to\infty\\ n\geq n_0}} \sqrt[n]{b_n} \le \limsup_{\substack{n\to\infty\\ n\geq n_0-k}} \sqrt[n]{b_{n+k}} =: l'. \label{e_rieder}
\end{equation}
Beweis. Angenommen, \[l' < l.\] Dann gibt es reelle Zahlen $s,t$, so dass \[l' < s < t < l.\] Da $t < l$, haben wir \[t < \sup\{\sqrt[n]{b_n} \mid n \ge m\}\] für alle $m\in\N$, $m \ge n_0$. Das heißt, für alle $m\in\N$ mit $m\ge n_0$ gibt es ein $n\in\N$, so dass $n\geq m$ und \[t < \sqrt[n]{b_n}.\] Mit anderen Worten: die Menge \[\{n \in \N \mid n \geq n_0, t < \sqrt[n]{b_n}\}\] ist unendlich. Da alle Glieder der Folge $(\sqrt[n]{b_{n+k}})$ nichtnegativ sind, gilt $0 \leq l'$, also $0 < t$. Wir setzen \[w := \log{t}.\] Aufgrund der (strengen) Monotonie der Exponentialfunktion folgt damit für alle $n\in\N$, $n\geq n_0$ aus \[\exp(w) = t < \sqrt[n]{b_n},\] dass \[0 < b_n \quad \text{und} \quad w < \frac1n \log{b_n}.\] Somit ist die Menge \[\{n\in\N\mid n+k\geq n_0,0 < b_{n+k},w<\frac1{n+k}\log(b_{n+k})\}\] unendlich. Für alle $n\in\N$ mit $n\geq n_0-k$ und $0 < b_{n+k}$ impliziert \[w<\frac1{n+k}\log(b_{n+k}),\] dass \[\frac{n+k}n w < \frac1n\log(b_{n+k}).\] Da $0<s$, existiert $v := \log s$. Die Monotonie der Exponentialfunktion liefert $v < w$. Folglich existiert ein $n_1 \in \N$, so dass für alle $n\in\N$ mit $n\geq n_1$: \[v < \frac{n+k}n w.\] Demnach sind die Mengen \[\{n\in\N\mid n+k\geq n_0, 0<b_{n+k},v<\frac1n\log(b_{n+k})\},\] und mithin \[\{n\in\N\mid n\geq n_0-k, s < \sqrt[n]{b_{n+k}}\},\] unendlich. Für alle $m\in\N$ mit $m\geq n_0-k$ ist also \[s < \sup\{\sqrt[n]{b_{n+k}} \mid n \geq m\},\] das heißt \[ s \leq \limsup_{n\to\infty} \sqrt[n]{b_{n+k}} = l' < s,\] was im Widerspruch zur Irreflexivität von $<$ steht. Somit gilt nicht $l' < l$, und wir schließen auf \eqref{e_rieder}. $\Box$
Korollar 1. Sei $(b_n)_{n\in\N}$ eine Folge in $[0,\infty)$ und $k\in \N_0$. Dann gilt \[l := \limsup_{n\to\infty} \sqrt[n]{b_n} = \limsup_{n\to\infty} \sqrt[n]{b_{n+k}} =: l'.\]
Beweis. Das Lemma liefert sofort \[l \leq l'.\] Sei jetzt $(b'_n)_{n\in\N}$ gegeben durch \[b'_n = b_{n+k}.\] Dann folgt aus dem Lemma, angewendet auf $(b'_n)$ anstelle von $(b_n)$ und $-k$ anstelle von $k$, \[l' = \limsup_{n\to\infty} \sqrt[n]{b'_n} \leq \limsup_{\substack{n\to\infty\\ n\geq 1-(-k)}} \sqrt[n]{b'_{n+(-k)}} =: l''.\] Für alle $n\in\N$ mit $n \geq 1-(-k) = 1+k$ gilt aber: \[b'_{n+(-k)} = b_{(n+(-k))+k} = b_n.\] Das heißt, \[l'' = \limsup_{\substack{n\to\infty\\ n\geq 1+k}} \sqrt[n]{b_n} = \limsup_{n\to\infty} \sqrt[n]{b_n} = l.\] Aufgrund der Antisymmetrie von $\leq$ ergibt sich \[l=l',\] was zu beweisen war. $\Box$
Korollar 2. Sei $(a_n)_{n\in\N}$ eine Folge in $\R$ und $k\in \N_0$. Dann gilt \eqref{e_main}.
Beweis. \eqref{e_main} folgt unmittelbar aus Korollar 1 für $(b_n) = (|a_n|)$. $\Box$

Montag, 16. Dezember 2013

Übungen zur Analysis I (nicht vertieft): Alternativer Zugang zu Blatt 8, Aufgabe 4 b)

In Blatt 8, Aufgabe 4 b) geht es darum zu zeigen, dass für alle Folgen reeller Zahlen $(a_n)_{n\in\N_0}$ und alle $k\in\N_0$ die Potenzreihen \[\sum_{n=0}^\infty a_n \quad \text{und} \quad \sum_{n=0}^\infty a_{n+k}\] denselben Konvergenzradius besitzen. Ein Beweis dieser Tatsache ergibt sich, indem man ausnutzt, dass der Konvergenzradius der Potenzreihe \[P(x) = \sum_{n=0}^\infty a_nx^n\] dasjenige eindeutig bestimmte $r \in [0,+\infty]$ ist, so dass für alle $x\in\R$ die Reihe $P(x)$ konvergiert, wenn $|x|<r$, und divergiert, wenn $|x|>r$. Dieser Zugang wird in den Übungen vorgestellt.

Ein alternativer Zugang zur Aufgabe ergibt sich vermittels der Formel von Cauchy-Hadamard. Durch die Cauchy-Hadamardsche Formel nämlich verwandelt sich die Aussage aus der Aufgabe in die Identität:
\begin{equation}
 \limsup_{n\to\infty} \sqrt[n]{|a_n|} = \limsup_{n\to\infty} \sqrt[n]{|a_{n+k}|}. \label{e_claim}
\end{equation} Im Folgenden geben wir, unter den Hypothesen der Aufgabe, einen Beweis von \eqref{e_claim}, der sich nicht auf die Interpretation der Folge $(a_n)_{n\in\N_0}$ als Potenzreihe stützt. Tatsächlich beweisen wir in mehreren Schritten einige weiter reichende Aussagen.
Lemma. Sei $(f_n)_{n\in\N}$ eine Folge von Funktionen $[0,+\infty)\to\R$, so dass für alle $d\geq1$ und alle $\epsilon>0$ ein $n_0\in\N$ existiert, so dass für alle $n\geq n_0$ und alle $x\in[0,d]$: \[|f_n(x)-x|<\epsilon.\] (Bemerkung: Diese Bedingung bedeutet, dass die Funktionenfolge $(f_n)$ auf $[0,+\infty)$ kompakt konvergent gegen $\mathrm{id}_{[0,+\infty)}$ ist.) Sei außerdem $(x_n)_{n\in\N}$ eine Folge in $[0,+\infty)$. Dann ist $h$ ein Häufungspunkt von $(f_n(x_n))_{n\in\N}$, wenn $h$ ein Häufungspunkt von $(x_n)_{n\in\N}$ ist.
Beweis. Angenommen also, $h$ ist ein Häufungspunkt von $(x_n)_{n\in\N}$ (insbesondere gilt $h\in\R$). Sei $\epsilon>0$. Dann ist \[d := \max(h+\frac\epsilon2,1) \geq 1 \quad \text{und} \quad \frac\epsilon2 > 0.\] Daher existiert ein $n_0 \in \N$, so dass für alle $n\geq n_0$ und alle $x\in[0,d]$: \[|f_n(x)-x|<\frac\epsilon2.\] Sei jetzt $x \in [0,+\infty)$ mit $|x-h|<\frac\epsilon2$. Dann folgt $x\in[0,d]$ und wir haben für alle $n\geq n_0$: \[|f_n(x)-h| \leq |f_n(x)-x|+|x-h| < \frac\epsilon2+\frac\epsilon2=\epsilon.\] Mit der Menge \[\{n\in\N \mid |x_n-h|<\frac\epsilon2\}\] ist also auch die Menge \[\{n\in\N \mid |f_n(x_n)-h|<\epsilon\}\] unendlich. Demzufolge handelt es sich bei $h$ um einen Häufungspunkt von $(f_n(x_n))_{n\in\N}$. $\Box$
Proposition. Sei $k\in\N_0$. Es bezeichne $(f_n)_{n\in\N}$ diejenige Folge von Funktionen $[0,+\infty)\to\R$, die für alle $n\in\N$ und alle $x\in[0,+\infty)$ durch \[f_n(x) = \sqrt[n]{x^{n+k}}\] gegeben ist. Dann gibt es für alle $d\geq1$ und alle $\epsilon>0$ ein $n_0\in\N$, so dass für alle $n\geq n_0$ und alle $x \in [0,d]$: \begin{equation}|f_n(x)-x|<\epsilon.\label{e_lemma2}\end{equation}
Beweis. Seien $d\geq 1$ und $\epsilon>0$ vorgegeben. Dann gilt \[\lim_{n\to\infty}\sqrt[n]{d^k}=\lim_{n\to\infty}\sqrt[n]{\epsilon^k}=1,\] das heißt es existiert ein $n_0\in\N$, so dass für alle $n\in\N$ mit $n\geq n_0$ gilt: \[|\sqrt[n]{d^k}-1|<\frac\epsilon{d} \quad \text{und} \quad |\sqrt[n]{\epsilon^k}-1|<\epsilon.\] Sei jetzt $x\in[0,d]$. Angenommen, $x\leq1$ und $x<\epsilon$. Dann gilt:
\begin{align*}
 |f_n(x)-x| &= |\sqrt[n]{x^{n+k}}-x| = |(\sqrt[n]x)^{n+k}+x| = |x(\sqrt[n]x)^k-x| \\ &= x|(\sqrt[n]x)^k-1| = x(1-\sqrt[n]{x^k}) \leq x < \epsilon.
\end{align*} Angenommen, $x\leq 1$ und $\epsilon\leq x$. Dann haben wir:
\begin{align*}
 |f_n(x)-x| = x(1-(\sqrt[n]x)^k) &\leq 1-(\sqrt[n]x)^k \\ &\leq 1-(\sqrt[n]\epsilon)^k = |\sqrt[n]{\epsilon^k}-1|<\epsilon.
\end{align*} Wenn weder $x\leq1$ und $x<\epsilon$ noch $\epsilon\leq x\leq 1$, dann gilt $1<x$. Es folgt
\begin{align*}
 |f_n(x)-x| = x((\sqrt[n]x)^k-1) \leq d((\sqrt[n]d)^k-1) < d\frac\epsilon{d} = \epsilon,
\end{align*} was zu beweisen war. $\Box$
Korollar 1. Sei $k\in\N_0$. Es bezeichne $(g_n)_{n\in\N}$ diejenige Folge von Funktionen $[0,+\infty)\to\R$, die für alle $n\in\N$ und alle $y\in[0,+\infty)$ durch \[g_n(y) = \sqrt[n+k]{y^n}\] gegeben ist. Dann gibt es für alle $d\geq1$ und alle $\epsilon>0$ ein $n_0\in\N$, so dass für alle $n\geq n_0$ und alle $y\in [0,d]$: \[|g_n(y)-y|<\epsilon.\]
Beweis. Seien $d\geq1$ und $\epsilon>0$ vorgegeben. Sei $(f_n)$ die Folge aus der Proposition. Dann existiert nach der Proposition ein $n_0\in\N$, so dass für alle $n\geq n_0$ und alle $y\in[0,d]$ die Beziehung \eqref{e_lemma2} gilt. Für alle $n\in\N$ gilt: Als Komposition monotoner Funktionen ist $g_n$ monoton; zudem ist wegen $d\geq1$: \[g_n(d) = \sqrt[n+k]{d^n} \leq d.\] Das heißt, für alle $y \in [0,d]$ gilt \[0 = g_n(0) \leq g_n(y) \leq g_n(d) \leq d,\] mit anderen Worten \[y \in [0,d].\] Wenn $n\geq n_0$, ist also \[|g_n(y)-y| = |y-g_n(y)| = |f_n(g_n(y))-g_n(y)| < \epsilon\] mit Hilfe von \eqref{e_lemma2}. $\Box$
Korollar 2. Sei $k\in\N_0$ und $(c_n)_{n\in\N}$ eine Folge in $[0,+\infty)$. Dann besitzen die Folgen \[(x_n) := (\sqrt[n+k]{c_n}) \quad \text{und} \quad (y_n) := (\sqrt[n]{c_n})\] dieselben eigentlichen und uneigentlichen Häufungspunkte.
Beweis. $(f_n)$ und $(g_n)$ mögen (respektive) wie in der Proposition und Korollar 1 erklärt sein. Dann gilt für alle $n\in\N$: \[f_n(x_n) = \sqrt[n]{c_n} = y_n.\] Nach der Proposition und dem Lemma ist jeder eigentliche Häufungspunkt von $(x_n)$ ein Häufungspunkt von $(y_n)$. Da \[g_n(y_n) = g_n(f_n(x_n)) = x_n,\] ist umgekehrt nach Korollar 1 und dem Lemma jeder eigentliche Häufungspunkt von $(y_n)$ ein Häufungspunkt von $(x_n)$.

$-\infty$ ist weder uneigentlicher Häufungspunkt von $(x_n)$ noch uneigentlicher Häufungspunkt von $(y_n)$, da alle Folgenglieder von $(x_n)$ und alle Folgenglieder von $(y_n)$ nichtnegativ (das heißt $\geq0$) sind; wäre etwa $-\infty$ Häufungspunkt (im uneigentlichen Sinn) von $(x_n)$, so würde die Unendlichkeit der Menge \[\{n\in\N \mid x_n < 0\}\] folgen.

Angenommen, $+\infty$ ist uneigentlicher Häufungspunkt von $(x_n)$. Sei $K \in \R$ beliebig. Dann gilt für alle $x \in \R$ mit $x > \max(K,1)$ und alle $n\in\N$: \[f_n(x) = \sqrt[n]{x^{n+k}} = (\sqrt[n]x)^{n+k} = x\cdot \sqrt[n]{x^k} > x\cdot 1 = x > K.\] Daher ist mit der Menge \[\{n\in\N \mid x_n > \max(K,1)\}\] auch die Menge \[\{n\in\N \mid f_n(x_n) > K\}\] unendlich. Folglich ist $+\infty$ uneigentlicher Häufungspunkt von $(y_n)$.

Angenommen umgekehrt, $+\infty$ ist uneigentlicher Häufungspunkt von $(y_n)$. Sei $L\in\R$ beliebig. Dann gilt für alle $y\in\R$ mit $y > \max(L^{1+k},1)$ und alle $n\in\N$: \[g_n(y) = \sqrt[n+k]{y^n} \geq \sqrt[1+k]y > L.\] Daher ist mit der Menge \[\{n\in\N \mid y_n > \max(L^{1+k},1)\}\] auch die Menge \[\{n\in\N \mid g_n(y_n) > L\}\] unendlich. Folglich ist $+\infty$ uneigentlicher Häufungspunkt von $(x_n)$. $\Box$
Korollar 3. Sei $k\in\N_0$ und $(b_n)_{n\in\N}$ eine Folge in $[0,+\infty)$. Dann besitzen die Folgen \[(\sqrt[n]{b_{n}}) \quad \text{und} \quad (\sqrt[n]{b_{n+k}})\] dieselben eigentlichen und uneigentlichen Häufungspunkte.
Beweis. Offensichtlich besitzen die Folgen \[(\sqrt[n]{b_{n}}) \quad \text{und} \quad (\sqrt[n+k]{b_{n+k}})\] dieselben eigentlichen und uneigentlichen Häufungspunkte. Nach Korollar 2 (angewendet auf die Folge $(c_n)$, die durch $c_n = b_{n+k}$ gegeben ist) besitzen zudem die Folgen \[(\sqrt[n+k]{b_{n+k}}) \quad \text{und} \quad (\sqrt[n]{b_{n+k}})\] dieselben eigentlichen und uneigentlichen Häufungspunkte. Somit folgt die Behauptung. $\Box$
Korollar 4. Sei $k\in\N_0$ und $(a_n)_{n\in\N}$ eine Folge in $\R$. Dann gilt \eqref{e_claim}.
Beweis. Bekanntlich ist der Limes superior einer Folge reeller Zahlen der größte eigentliche oder uneigentliche Häufungspunkt dieser Folge (wobei die typische Ordnung auf $\R \cup \{\pm\infty\}$ verwendet wird, wonach $-\infty<x<+\infty$ für alle $x\in\R$ gilt). Daher ist der Limes superior der Folge $(\sqrt[n]{|a_n|})$ gleich dem größten Häufungspunkt von $(\sqrt[n]{|a_n|})$. Nach Korollar 3 (angewendet auf die Folge $(b_n)$, die durch $b_n = |a_n|$ gegeben ist) ist der größte Häufungspunkt von $(\sqrt[n]{|a_n|})$ gleich dem größten Häufungspunkt von $(\sqrt[n]{|a_{n+k}|})$. Der größte Häufungspunkt von $(\sqrt[n]{|a_{n+k}|})$ ist aber wiederum gleich dem Limes superior von $(\sqrt[n]{|a_{n+k}|})$. $\Box$

Dienstag, 10. Dezember 2013

Übungen zur Analysis I (nicht vertieft): Lösungen zu Blatt 8

Aufgabe 5 (Staatsexamen Frühjahr 2013, Thema Nr. 1, Aufgabe 1 b)).
Für welche $x \in \mathbb R$ konvergiert die folgende Reihe? Berechnen Sie gegebenenfalls den Grenzwert.
\begin{equation}\label{e_series0}
 \sum_{n=1}^\infty nx^{n-1}.
\end{equation}
Lösung. Sei $x \in \mathbb R$. Angenommen, $|x|<1$. Dann ist die geometrische Reihe \[\sum_{n=0}^\infty x^n\] bekanntlich absolut konvergent, und zwar gegen $\frac1{1-x}$. Dem Cauchyschen Produktsatz zufolge ist also die Reihe \[\sum_{n=0}^\infty\left(\sum_{k=0}^nx^kx^{n-k}\right) = \sum_{n=0}^\infty\left(\sum_{k=0}^nx^n\right) = \sum_{n=0}^\infty(n+1)x^n\] (absolut) konvergent und besitzt die Summe \[\left(\frac1{1-x}\right)^2.\] Die Reihe \eqref{e_series0} entsteht aus der Reihe \[\sum_{n=0}^\infty(n+1)x^n\] durch Indexverschiebung. Genauer gesagt gilt für alle $N \in \mathbb N$: \[\sum_{n=0}^{N-1}(n+1)x^n = \sum_{n=1}^{N}nx^{n-1}.\] Mithin ist die Reihe \eqref{e_series0} konvergent gegen $\left(\frac1{1-x}\right)^2$. Angenommen, $|x| \geq 1$ . Dann gilt für alle $n \in \mathbb N$: \[|nx^{n-1}| = n|x|^{n-1} \geq n1 \geq 1.\] Insbesondere ist die Folge \[(nx^{n-1})_{n \in \mathbb N}\] keine Nullfolge. Die Reihe \eqref{e_series0} ist daher nicht konvergent.
Aufgabe 6 (Staatsexamen Frühjahr 2013, Thema Nr. 2, Aufgabe 1).
Bestimmen Sie für jede der folgenden Reihen alle $x \in \mathbb R$, für die die Reihe konvergiert.
\[
 \sum_{k=0}^\infty e^{xk}, \quad \sum_{k=0}^\infty e^kx^k, \quad \sum_{k=0}^\infty \frac1{e^x + k}.
\]
Lösung. Sei $x \in \mathbb R$.
(i). Wir wissen: für alle $q \in \mathbb R$ ist die geometrische Reihe \[\sum_{k=0}^\infty q^k\] genau dann konvergent, wenn $|q|<1$. Daher ist die Reihe \[\sum_{k=0}^\infty e^{xk} = \sum_{k=0}^\infty (e^x)^k\] genau dann konvergent, wenn \[e^x = |e^x| < 1;\] hier geht ein, dass $0 < e^x$. Aufgrund der strikten Monotonie der Exponentialfunktion gilt aber $e^x < 1 = e^0$ genau dann, wenn $x<0$.
(ii). Die Reihe \[\sum_{k=0}^\infty e^kx^k = \sum_{k=0}^\infty (ex)^k\] ist genau dann konvergent, wenn \[e|x| = |ex| < 1.\] Letzteres ist gleichbedeutend damit, dass $|x|<\frac1e$.
(iii). Nach dem archimedischen Axiom gibt es eine natürliche Zahl $l$, so dass $e^x < l$. Für alle $k \in \mathbb N_0$ gilt also \[e^x + k < l + k,\] das heißt, \[0 < \frac1{l+k} < \frac1{e^x + k}.\] Da die harmonische Reihe (bestimmt gegen $+\infty$) divergent ist, ist auch die Reihe \[\sum_{k=0}^\infty \frac1{l+k}\] (bestimmt gegen $+\infty$) divergent, denn für alle $K \in \mathbb N_0$ gilt: \[\sum_{k=0}^K \frac1{l+k} = \sum_{n=1}^{l+K}\frac1n - \sum_{n=1}^{l-1}\frac1n.\] Nach dem Minorantenkriterium ist auch die Reihe \[\sum_{k=0}^\infty \frac1{e^x + k}\] (bestimmt gegen $+\infty$) divergent.
Aufgabe 7 (Staatsexamen Herbst 2012, Thema Nr. 1, Aufgabe 1).
Sei
\begin{equation}
 \sum_{n=0}^\infty a_nx^n \label{e_7}
\end{equation}
eine Potenzreihe mit Konvergenzradius $r$, wobei $1 < r < \infty$ sei.
a) Bestimmen Sie den Konvergenzradius der Potenzreihe
\begin{equation}\label{e_7a}
 \sum_{n=0}^\infty a_nx^{2n}.
\end{equation}
b) Bestimmen Sie den Konvergenzradius der Potenzreihe
\begin{equation}\label{e_7b}
 \sum_{n=0}^\infty c_nx^n,
\end{equation}
wobei $c_n = a_n^n$ für alle $n \in \mathbb N_0$ sei.
Lösung. a) Sei $x \in \mathbb R$. Angenommen, $|x|<\sqrt{r}$. Dann gilt: \[|x^2|=|x|^2<(\sqrt{r})^2=r.\] Da $r$ der Konvergenzradius von \eqref{e_7} ist, ist demnach die Reihe \[\sum_{n=0}^\infty a_n(x^2)^n = \sum_{n=0}^\infty a_nx^{2n}\] absolut konvergent. Angenommen, $|x|>\sqrt{r}$. Dann gilt $|x^2|>r$. Da $r$ der Konvergenzradius von \eqref{e_7} ist, ist die Reihe \[\sum_{n=0}^\infty a_n(x^2)^n = \sum_{n=0}^\infty a_nx^{2n}\] divergent. Diesen Überlegungen zufolge ist der Konvergenzradius der Reihe \eqref{e_7a} gleich $\sqrt{r}$.

b) Nach der Formel von Cauchy-Hadamard gilt \[\frac1{\limsup_{n\to\infty}\sqrt[n]{|a_n|}} = r,\] also \[\limsup_{n\to\infty}\sqrt[n]{|a_n|} = \frac1r.\] Zudem ist $\frac1r<1$, da $1<r$. Sei \[b := \frac12(\frac1r+1).\] Dann haben wir \[\frac1r < b < 1.\] Da \[\lim_{n \to \infty}\sup\{\sqrt[m]{|a_m|}\mid m \in \mathbb N, n \leq m\} = \frac1r,\] gibt es ein $n_0 \in \mathbb N$, so dass für alle $n\in\mathbb N$ mit $n \geq n_0$: \[\sup\{\sqrt[m]{|a_m|}\mid m \in \mathbb N,n \leq m\} < b.\] Insbesondere haben wir für alle $m\in\mathbb N$, $m \geq n_0$: \[\sqrt[m]{|a_m|} < b,\] und somit \[|a_m| < b^m.\] Wegen $0<b<1$ ist \[\lim_{m\to\infty}b^m=0,\] also auch \[\lim_{m\to\infty} |a_m|=0.\] Das heißt, \[\limsup_{n\to\infty}\sqrt[n]{|c_n|} = \lim_{n\to\infty}\sqrt[n]{|a_n^n|}=\lim_{n\to\infty}\sqrt[n]{|a_n|^n}=\lim_{n\to\infty}|a_n|=0.\] Der Konvergenzradius der Reihe \eqref{e_7b} beträgt also, gemäß Cauchy-Hadamardscher Formel, $+\infty$.
Aufgabe 8 (Staatsexamen Frühjahr 2012, Thema Nr. 2, Aufgabe 3).
a) Bestimmen Sie den Konvergenzradius $r$ der Potenzreihe
\[
 R(x) = \sum_{n=0}^\infty (-1)^n \frac1{\sqrt{3^n}(5n^2 + 1)} x^n.
\]
b) Beurteilen Sie, ob $R(x)$ an den Stellen $x = r$ und $x = -r$ konvergiert oder divergiert.
Lösung. a) Für alle $n \in \mathbb N$ haben wir:
\begin{align*}
 \sqrt[n]{\left|(-1)^n\frac1{\sqrt{3^n}(5n^2+1)}\right|} & = \sqrt[n]{\frac1{\sqrt{3^n}(5n^2+1)}} \\
 & = \frac1{\sqrt[n]{\sqrt{3^n}(5n^2+1)}} \\
 & = \frac1{\sqrt[n]{\sqrt{3^n}}\sqrt[n]{5n^2+1}} \\
 & = \frac1{\sqrt{\sqrt[n]{3^n}}\sqrt[n]{5n^2+1}} \\
 & = \frac1{\sqrt3\sqrt[n]{5n^2+1}},
\end{align*}
zudem
\[
 \sqrt[n]{5n^2+1} = \sqrt[n]{\left(5+\frac1{n^2}\right)n^2} = \sqrt[n]{5+\frac1{n^2}}\sqrt[n]{n^2} = \sqrt[n]{5+\frac1{n^2}}(\sqrt[n]n)^2
\]
und
\[
 \sqrt[n]5 < \sqrt[n]{5+\frac1{n^2}} \leq \sqrt[n]6.
\]
Da
\[
 \lim_{n\to\infty}\sqrt[n]5 = \lim_{n\to\infty}\sqrt[n]{6} = 1,
\]
haben wir also
\[
 \lim_{n\to\infty}\sqrt[n]{5+\frac1{n^2}} = 1.
\]
Und da
\[
 \lim_{n\to\infty}\sqrt[n]n = 1,
\]
ergibt sich weiter
\[
 \lim_{n\to\infty}\sqrt[n]{5n^2+1} = 1 \cdot 1^2 = 1.
\]
Demnach gilt
\[
 \lim_{n\to\infty}\sqrt[n]{\left|(-1)^n\frac1{\sqrt{3^n}(5n^2+1)}\right|} = \frac1{\sqrt3},
\]
und mithin
\[
 \limsup_{n\to\infty}\sqrt[n]{\left|(-1)^n\frac1{\sqrt{3^n}(5n^2+1)}\right|} = \frac1{\sqrt3}.
\]
Die Formel von Cauchy-Hadamard liefert also
\[
 r = \frac1{\frac1{\sqrt3}} = \sqrt3.
\]

b) Es gilt
\begin{align}
 R(\pm r) & = R(\pm\sqrt3) = \sum_{n=0}^\infty (-1)^n\frac1{\sqrt{3^n}(5n^2+1)}(\pm\sqrt3)^n \notag \\ & = \sum_{n=0}^\infty (\mp1)^n\frac1{5n^2+1}, \label{e_8b}
\end{align}
und für alle $n \in \mathbb N$ haben wir
\[
 0 < n^2 < 5n^2 + 1,
\]
also
\[
 \left|(\mp1)^n\frac1{5n^2+1}\right| = \frac1{5n^2+1}  < \frac1{n^2}.
\]
Die Reihe
\[
 \sum_{n=1} \frac1{n^2}
\]
ist bekanntlich konvergent (gegen $\frac{\pi^2}6$). Nach dem Majorantenkriterium sind daher die Reihen \eqref{e_8b} absolut konvergent, mithin konvergent (im gewöhnlichen Sinn).

Übungen zur Analysis I (nicht vertieft): Lösungen zu Blatt 5

Aufgabe 5 (Staatsexamen Frühjahr 2012, Thema Nr. 1, Aufgabe 1).
a) Sei $(a_n)_{n \in \mathbb N}$ eine gegen $a$ konvergente Folge in $\mathbb R$. Zeigen Sie, dass dann auch die Folge $(b_n)_{n \in \mathbb N}$ mit
\[
 b_n := \frac12(a_n + a_{n+1}) \qquad \text{für alle } n \in \mathbb N
\]
gegen $a$ konvergiert.
b) Finden Sie eine Folge $(a_n)_{n \in \mathbb N}$, die nicht konvergiert, so dass die zugehörige Folge $(b_n)_{n \in \mathbb N}$ konvergiert.
c) Sei vorausgesetzt, dass $(a_n)_{n \in \mathbb N}$ monoton wächst und dass $(b_n)_{n \in \mathbb N}$ konvergiert. Zeigen Sie, dass dann auch $(a_n)_{n \in \mathbb N}$ konvergiert.
Lösung. a) Sei $\epsilon \in \mathbb R$, $\epsilon > 0$. Da die Folge $(a_n)_{n \in \mathbb N}$ konvergent gegen $a$ ist, existiert ein $n_0 \in \mathbb N$, so dass für alle $n \in \mathbb N$ mit $n \geq n_0$ gilt: \[|a_n - a| < \epsilon,\] das heißt \[-\epsilon < a_n-a < \epsilon.\] Sei jetzt $n \in \mathbb N$ und $n \geq n_0$. Angenommen, $a_n \leq a_{n+1}$. Dann folgt \[a_n = \frac12(a_n+a_n) \leq \frac12(a_n+a_{n+1}) \leq \frac12(a_{n+1}+a_{n+1}) = a_{n+1},\] also \[-\epsilon < a_n-a \leq b_n-a \leq a_{n+1}-a < \epsilon,\] und somit \[|b_n-a|<\epsilon.\] Wenn nicht $a_n \leq a_{n+1}$, so ist $a_{n+1}<a_n$, und durch ein entsprechendes Argument gelangt man zum selben Schluss: $|b_n-a|<\epsilon$. Die Folge $(b_n)_{n\in\mathbb N}$ ist daher konvergent gegen $a$, q. e. d. $\Box$

b) Wenn $a_n = (-1)^n$ für alle $n \in \mathbb N$, dann ist die Folge $(a_n)_{n \in \mathbb N}$ nicht konvergent (denn: \[|a_{n+1}-a_n| = |(-1)^n((-1)-1)| = 2\] für alle $n \in \mathbb N$, das heißt $(|a_{n+1}-a_n|)$ ist keine Nullfolge ...). Andererseits gilt für alle $n \in \mathbb N$: \[b_n = \frac12((-1)^n+(-1)^{n+1}) = \frac12(-1)^n(1+(-1)) = 0,\] das heißt die Folge $(b_n)_{n\in\mathbb N}$ ist konvergent (gegen $0$).

c) Da $(b_n)_{n\in\mathbb N}$ konvergiert, ist $(b_n)$ insbesondere (nach oben) beschränkt, das heißt es gibt eine reelle Zahl $c$, so dass für alle $n \in \mathbb N$ gilt $b_n \leq c$. Aufgrund der Monotonie von $(a_n)$ ist, für alle $n\in\mathbb N$, $a_n \leq a_{n+1}$, also \[a_n \leq \frac12(a_n+a_{n+1}) = b_n \leq c.\] Das heißt die Folge $(a_n)$ ist nach oben beschränkt. Folglich ist $(a_n)$ konvergent als nach oben beschränkte, monoton wachsende Folge. $\Box$
Aufgabe 6 (Staatsexamen Herbst 2007, Thema Nr. 1, Aufgabe 1).
Gegeben sei die Folge $(a_n)_{n \in \mathbb N}$ mit
\[
 a_1 = 1, \qquad a_{n+1} = \sqrt{12 + a_n}, \quad n \geq 1.
\]
a) Zeigen Sie, dass $(a_n)$ monoton wachsend und beschränkt ist.
b) Bestimmen Sie den Grenzwert von $(a_n)$.
Lösung. a) Es gilt \[a_{1+1} = \sqrt{12+a_1} = \sqrt{13},\] das heißt \[1 \leq a_1 \leq a_{1+1} \leq 4.\] Sei $n \in \mathbb N$ und gelte \[1 \leq a_n \leq a_{n+1} \leq 4.\] Dann folgt: \[1 < 13 \leq 12+a_n \leq 12+a_{n+1} \leq 16 = 4^2,\] und mithin \[1 < \sqrt{12+a_n} \leq \sqrt{12+a_{n+1}} \leq 4,\] also \[1 \leq a_{n+1} \leq a_{(n+1)+1} \leq 4.\] Aus dem Induktionsprinzip ergibt sich, dass für alle $n \in \mathbb N$ gilt: \[1 \leq a_n \leq a_{n+1} \leq 4.\] Insbesondere haben wir für alle $n \in \mathbb N$: \[a_n \leq a_{n+1}\] sowie \[1 \leq a_n \leq 4.\] Das heißt, die Folge $(a_n)$ ist monoton wachsend und beschränkt. $\Box$

b) Nach Teil a) ist $(a_n)$, als monoton wachsende, nach oben beschränkte Folge, konvergent. Es gibt also eine reelle Zahl $a$, so dass \[\lim_{n\to\infty}a_n = a.\] Daher gilt auch \[\lim_{n\to\infty}a_{n+1}=a.\] Da, für alle $n \in \mathbb N$, \[a_{n+1}^2=12+a_n,\] folgt weiter \[a^2 = \lim_{n\to\infty} a_{n+1}^2 = \lim_{n\to\infty} (12+a_n) = 12+a,\] also \[a^2-a-12=(a+3)(a-4)=0.\] Das heißt $a=-3$ oder $a=4$. Da für alle $n \in \mathbb N$ gilt \[1 \leq a_n\] (siehe a)), folgt \[1 \leq a.\] Da nicht $1 \leq -3$, schließen wir, dass $a=4$.
Aufgabe 7 (Staatsexamen Herbst 2001, Thema Nr. 3, Aufgabe 1)Die Folge $(a_n)_{n \in \mathbb N}$ sei definiert durch
\[
 a_1 = 2, \quad a_{n+1} = \frac1{a_1 + 2a_2 + \dots + na_n}.
\]
Zeigen Sie:
a) $(a_n)$ ist monoton fallend und beschränkt.
b) $\lim_{n \to \infty} a_n = 0$.
Lösung. a) Es gilt: \[a_2=a_{1+1}=\frac1{a_1}=\frac12.\] Sei $n \in \mathbb N$, $n \geq 2$, so dass, für alle $j \in \mathbb N$ mit $2 \leq j \leq n$, gilt \[a_j = \frac1j.\] Dann haben wir \begin{align*}a_{n+1}&=\frac1{a_1 + 2a_2 + \dots + na_n} = \frac1{2+2\frac12+\dots+n\frac1n} \\ &=\frac1{2+\underbrace{1+\dots+1}_{(n-1)\text{-mal}}}=\frac1{2+(n-1)}=\frac1{n+1}.\end{align*} Das heißt, für alle $j \in \mathbb N$ mit $2\leq j\leq n+1$, gilt: \[a_j=\frac1j.\] Das Induktionsprinzip besagt nun, dass, für alle $n \in \mathbb N$ mit $n \geq 2$: \[a_n=\frac1n.\] Demnach ist \[0 < a_{n+1} < a_n < 1\] für alle $n \in \mathbb N$ mit $n \geq 2$. Zudem gilt \[a_{1+1} = a_2 = \frac12 < 2 = a_1.\] Die Folge $(a_n)$ ist somit (streng) monoton fallend und beschränkt. $\Box$

b) Bekanntlich gilt \[\lim_{n \to \infty} \frac1n = 0.\] Nach Teil a) stimmen die Folgen $(\frac1n)_{n\in\mathbb N}$ und $(a_n)_{n \in \mathbb N}$ für $n \geq 2$ überein. Somit konvergiert $(a_n)$ ebenfalls gegen $0$. $\Box$
Aufgabe 8. Untersuchen Sie die Folge $(a_n)_{n \in \mathbb N}$ auf Konvergenz, wenn für alle $n \in \mathbb N$ gilt:
\[
\text{a)} \; a_n = \sqrt[n]{n^2 + n}. \quad \text{b)} \; a_n = \frac{n!}{n^n}. \quad \text{c)} \; a_n = \sqrt[n]{a^n + b^n},
\] wobei $a,b \in [0,\infty)$. 
Lösung. a) Für alle $n \in \mathbb N$ gilt: \begin{align*}a_n &= \sqrt[n]{n^2 + n} = \sqrt[n]{n^2\left(1 + \frac1n\right)} \\ &= \sqrt[n]{n^2}\sqrt[n]{1+\frac1n} = (\sqrt[n]n)^2\sqrt[n]{1+\frac1n}.\end{align*} Wir wissen, dass \[\lim_{n\to\infty}\sqrt[n]n = 1.\] Außerdem gilt für alle $n \in \mathbb N$: \[0<\frac1n \leq 1,\] also \[1 < 1+\frac1n \leq 2,\] also \[1 < \sqrt[n]{1+\frac1n} \leq \sqrt[n]2.\] Da \[\lim_{n\to\infty}\sqrt[n]2 = 1,\] folgt \[\lim_{n\to\infty} \sqrt[n]{1+\frac1n} = 1\] und damit \[\lim_{n\to\infty} a_n = 1^2\cdot 1 = 1.\]

b) Nach der Bernoullischen Ungleichung gilt \[2 = 1+n\frac1n \leq (1+\frac1n)^n\] für alle $n \in \mathbb N$, und daher auch \[\frac{a_{n+1}}{a_n} = \frac{n^n}{n!}\frac{(n+1)!}{(n+1)^{n+1}} = \frac{n^n}{(n+1)^n} = \frac1{\left(1+\frac1n\right)^n} \leq \frac12.\] Hieraus folgert man induktiv, dass, für alle $n \in \mathbb N$: \[0 < a_n \leq \left(\frac12\right)^{n-1}a_1.\] Mit \[\lim_{n\to\infty} \left(\frac12\right)^n = 0\] ist also auch \[\lim_{n\to\infty} a_n = 0.\]

c) Angenommen, $a<b$. Dann ist $0 < b$ und für alle $n\in\mathbb N$ gilt:
\begin{align*}
 a_n &= \sqrt[n]{a^n+b^n}= \sqrt[n]{b^n(\frac{a^n}{b^n}+1)} = \sqrt[n]{b^n}\sqrt[n]{\frac{a^n}{b^n}+1} \\ &= b\sqrt[n]{\frac{a^n}{b^n}+1}
\end{align*}
sowie
\[
 0 \leq a^n < b^n,
\]
das heißt
\[
 1 = 0+1 \leq \frac{a^n}{b^n}+1 \leq 1+1 = 2,
\]
also
\[
 1 \leq \sqrt[n]{\frac{a^n}{b^n}+1} \leq \sqrt[n]2.
\]
Da
\[
 \lim_{n\to\infty} \sqrt[n]2 = 1,
\]
folgt
\[
 \lim_{n\to\infty} \sqrt[n]{(\frac{a}{b})^n+1} = 1,
\]
und somit:
\[
 \lim_{n\to\infty} a_n = b \cdot 1 = b.
\]
Angenommen, $b<a$. Dann ergibt sich analog (oder aus Symmetriegründen), dass $(a_n)$ konvergent gegen $a$ ist. Ist weder $a<b$ noch $b<a$, so gilt $a=b$ und wir haben (für alle $n \in \mathbb N$):
\[
 a_n = \sqrt[n]{2a^n} = \sqrt[n]2\sqrt[n]{a^n} = \sqrt[n]2\cdot a \longrightarrow 1 \cdot a = a
\]
für $n\to\infty$.

Montag, 10. Juni 2013

Übungen zur Geometrie: Lösungen zu Blatt 6

Aufgabe 4

Satz. Seien $X$ und $Y$ disjunkte, wegzusammenhängende topologische Räume, $A \subset X$ eine nichtleere Teilmenge, $f \colon A \to Y$ eine Abbildung. Dann ist der Raum $Z := Y \cup_f X$ wegzusammenhängend.
Bemerkung. In der Aufgabenstellung fehlt die Voraussetzung, dass $A \neq \emptyset$. Ohne diese zusätzliche Voraussetzung ist der Schluss auf den Wegzusammenhang von $Z$ jedoch offensichtlich nicht gültig.
Beweis. Seien $z_0,z_1 \in Z$. Als Äquivalenzklassen einer Äquivalenzrelation auf $Y \cup X$ gilt \[\emptyset \neq z_0,z_1 \subset Y \cup X.\] Es existieren also $s_0 \in z_0$ und $s_1 \in z_1$, und wir haben \[s_0,s_1 \in Y \cup X.\] Angenommen $s_0,s_1 \in X$. Dann gibt es einen Weg $\gamma$ in $X$ von $s_0$ nach $s_1$, da $X$ wegzusammenhängend ist. Die Komposition von $\gamma$ mit der evidenten – und stetigen – Abbildung \[X \to Y \cup X \to Y \cup_f X = Z\] ist folglich ein Weg in $Z$ von $[s_0] = z_0$ nach $[s_1] = z_1$. Analog argumentiert man für den Fall, dass $s_0,s_1 \in Y$. Angenommen nun $s_0 \in X$ und $s_1 \in Y$. Da $A \neq \emptyset$, gibt es ein Element $a \in A$. Aufgrund des Wegzusammenhangs von $X$ existiert ein Weg $\gamma$ in $X$ von $s_0$ nach $a$. Aufgrund des Wegzusammenhangs von $Y$ existiert ein Weg $\delta$ in $Y$ von $f(a)$ nach $s_1$. Da \[[a] = [f(a)]\] in $Z$, ist die Verkettung der Bildwege von $\gamma$ und $\delta$ in $Z$ ein Weg in $Z$ von $[s_0]=z_0$ nach $[s_1]=z_1$. Trifft keiner der bereits behandelten Fälle zu, so ist $s_0 \in Y$ und $s_1 \in X$. Analog zur vorherigen Situation erhält man einen Weg in $Z$ von $z_0$ nach $z_1$. $\Box$

Dienstag, 4. Juni 2013

Übungen zur Geometrie: Lösungen zu Blatt 5

Aufgabe 4


Zur Lösung zeigen wir den folgenden
Satz. Sei $n$ eine natürliche Zahl und $x \in \mathbb P_n(\mathbb R)$. Dann ist $\mathbb P_{n-1}(\mathbb R)$ homotopieäquivalent zu $\mathbb P_n(\mathbb R) \backslash \{x\}$.
Beweis. Nach Definition des projektiven Raums ist \[x^* := x \cup \{0\}\] ein eindimensionaler linearer Unterraum von $\mathbb R^{n+1}$. Daher existiert ein injektiver Homomorphismus \[\iota \colon \mathbb R^n \to \mathbb R^{n+1},\] so dass \begin{equation}\mathbb R^{n+1} = E \oplus x^*, \label{e_splitting}\end{equation} wobei \[E := \mathrm{im}(\iota).\] Aufgrund der Linearität von $\iota$ gilt \[\iota(\lambda \cdot v) = \lambda \cdot \iota(v)\] für alle reellen Zahlen $\lambda$ und alle $v \in \mathbb R^n$. Zudem ist $\mathrm{ker}(\iota) = \{0\}$. Demzufolge existiert eine, und nur eine, Abbildung \[i \colon \mathbb P_{n-1}(\mathbb R) \to \mathbb P_n(\mathbb R),\] so dass, für alle $v \in \mathbb R^n \backslash \{0\}$: \[i([v]) = [\iota(v)].\] Da $\iota$ (als lineare Abbildung) stetig ist, ist auch $i$ stetig (gemäß der universellen Eigenschaft der Quotiententopologie auf $\mathbb P_{n-1}(\mathbb R)$). Da \[(E \backslash \{0\}) \cap x = \emptyset,\] ist $i$ sogar eine – wiederum stetige – Abbildung \[i \colon \mathbb P_{n-1}(\mathbb R) \to \mathbb P_n(\mathbb R) \backslash \{x\}.\] Wir schreiben \[\pi \colon \mathbb R^{n+1} \to \mathbb R^n\] für die durch \eqref{e_splitting} gegebene Projektion auf $E$; das heißt, $\pi$ ist die eindeutig bestimmte lineare Abbildung, für die \[\pi \circ \iota = \mathrm{id}_{\mathbb R^n}\] sowie \[\mathrm{ker}(\pi) = x^*\] gilt. Analog dem Argument für $i$ oben existiert nun eine, und nur eine, Abbildung \[p \colon \mathbb P_n(\mathbb R) \backslash \{x\} \to \mathbb P_{n-1}(\mathbb R),\] so dass, für alle $w \in \mathbb R^{n+1} \backslash x^*$: \[p([w]) = [\pi(w)].\] Für alle $v \in \mathbb R^n \backslash \{0\}$ gilt: \[p \circ i ([v]) = p([\iota(v)]) = [\pi\circ\iota(v)] = [v].\] Das heißt wir haben \[p \circ i = \mathrm{id}_{\mathbb P_{n-1}(\mathbb R)}\] und mithin \[p \circ i \simeq \mathrm{id}_{\mathbb P_{n-1}(\mathbb R)}.\] Definieren wir jetzt \[F \colon \mathbb R^{n+1} \times I \to \mathbb R^{n+1}\] durch die Vorschrift \[F(w,t) = t \cdot \iota\circ \pi(w) + (1-t) \cdot w.\] Dann gilt \[\pi(F(w,t)) = \pi(w)\] für alle $w \in \mathbb R^{n+1}$ und alle $t \in I$. Das heißt durch Einschränkung von $F$ erhalten wir eine Abbildung \[(\mathbb R^{n+1}\backslash x^*) \times I \to \mathbb R^{n+1}\backslash x^*;\] dabei beachte man, dass $x^* = \mathrm{ker}(\pi)$. Zudem gilt \[F(\lambda\cdot w,t) = \lambda \cdot F(w,t)\] für alle reellen Zahlen $\lambda$ und alle $w$ und $t$ wie zuvor. Demnach existiert eine, und nur eine, Abbildung \[\overline F \colon (\mathbb P_n(\mathbb R) \backslash \{x\}) \times I \to \mathbb P_n(\mathbb R) \backslash \{x\},\] so dass, für alle $w \in \mathbb R^{n+1} \backslash x^*$ und alle $t \in I$: \[\overline F([w],t) = [F(w,t)].\] Die Abbildung $F$ ist offensichtlich stetig – damit auch ihre genannte Einschränkung. Aus der Definition der Quotiententopologie folgt die Stetigkeit von $\overline F$. Weiterhin gilt \[\overline F(-,0) = \mathrm{id}_{\mathbb P_n(\mathbb R) \backslash \{x\}}\] und \[\overline F(-,1) = i \circ p.\] Die Abbildung $\overline F$ ist also eine Homotopie zwischen der identischen Abbildung auf $\mathbb P_n(\mathbb R) \backslash \{x\}$ und der Abbildung $i \circ p$. Das heißt es gilt \[\mathrm{id}_{\mathbb P_n(\mathbb R) \backslash \{x\}} \simeq i \circ p.\] In der Folge haben wir \[\mathbb P_{n-1}(\mathbb R) \simeq_{\mathrm{h}} \mathbb P_n(\mathbb R) \backslash \{x\},\] was zu beweisen war. $\Box$
Bemerkung. Das Argument funktioniert ohne Weiteres auch für den Fall, dass $n = 0$. Frage: Was ist $\mathbb P_{-1}(\mathbb R)$? Was $\mathbb P_0(\mathbb R)$?

Mittwoch, 15. Mai 2013

Übungen zur Geometrie: Lösungen zu Blatt 4

Aufgabe 1


Sei $n$ eine natürliche Zahl $\neq 0$. Wir setzen \[e := (1,0,\dots,0) \in \mathbb R^{n+1}, \qquad e' := (1,0,\dots,0) \in \mathbb R^n\] und definieren Abbildungen \[f_\pm \colon D^n \to S^n\] durch \[ f_\pm(x_0,\dots,x_{n-1}) = (x_0,\dots,x_{n-1},\pm\sqrt{1-x_0^2-\dots-x_{n-1}^2});\] wobei $D^n$ wie üblich für die $n$-dimensionale Scheibe steht, das heißt: \[D^n = \{a \in \mathbb R^n:a_0^2+\dots+a_{n-1}^2 \le 1\}.\] Sei $x = (x_0,\dots,x_n) \in S^n$. Wir setzen \[x' := (x_0,\dots,x_{n-1})\] und definieren eine Abbildung $\gamma \colon I \to \mathbb R^n$ durch \[\gamma(t) = (1-t)x' + te'.\] Dann ist $\gamma$ offensichtlich ein Weg in $D^n$ von $x'$ nach $e'$. Ist nun $x_n \ge 0$ (resp. $x_n < 0$), so ist die Komposition $f_+ \circ \gamma$ (resp. $f_- \circ \gamma$) ein Weg in $S^n$ von $f_+(x') = x$ (resp. $f_-(x')=x$) nach $f_+(e') = f_-(e') = e$. Man beachte, dass die $f_\pm$ stetig sind. Ist zusätzlich zu $x$ noch ein weiterer Punkt $y \in S^n$ gegeben, so verbindet man $x$ und $y$, indem man zunächst den konstruierten Weg von $x$ nach $e$ und anschließend den entsprechenden Weg $y$ nach $e$ rückwärts läuft. $\Box$
Bemerkung. Obwohl in der Aufgabe lediglich gefordert ist, zu zeigen, dass die Sphären $S^n$ mit $n \ge 2$ wegzusammenhängend sind, ist die eindimensionale Sphäre $S^1$ selbstverständlich auch wegzusammenhängend (wie obiges Argument zeigt). Die $0$-Sphäre ist offenkundig nicht (weg-)zusammenhängend, da sie aus den beiden Punkten $-1$ und $1$ versehen mit diskreter Topologie besteht.

Aufgabe 2


Schritt 1. $\widehat{(X,\tau)} = (\widehat X, \widehat \tau)$ ist definitionsgemäß genau dann ein topologischer Raum, wenn $\widehat\tau$ eine Topologie auf $\widehat X$ ist. Wir weisen dies nach. Offensichtlich ist $\widehat\tau \subset \mathcal P(\widehat X)$. Zudem gilt $\emptyset \in \tau$, also $\emptyset \in \widehat\tau$, da $\tau \subset \widehat\tau$. Da $\emptyset$ kompakt und abgeschlossen in $(X,\tau)$ ist, haben wir $\widehat X = \widehat X \backslash \emptyset \in \widehat \tau$. Sei $(U_i)_{i\in I}$ eine Familie von Elementen von $\hat \tau$ und \[U = \bigcup_{i\in I}U_i.\] Wir definieren $I'$ als die Menge aller $i \in I$, so dass $U_i \in \tau$, und setzen $I'' := I \backslash I'$. Wenn $I'' = \emptyset$, so gilt $I' = I$ und $(U_i)$ ist eine Familie von Elementen von $\tau$; folglich ist $U \in \tau$, da $\tau$ eine Topologie ist (nach Voraussetzung). Angenommen nun, $I'' \neq \emptyset$. Für $i\in I''$ setzen wir $K_i := \widehat X \backslash U_i$. Mit \[K := \bigcap_{i\in I'}(X \backslash U_i) \cap \bigcap_{i\in I''}K_i \] gilt dann: \begin{align*}U & = \bigcup_{i\in I'} U_i \cup \bigcup_{i\in I''}U_i \\ & = (X \backslash \bigcap_{i\in I'}(X\backslash U_i))  \cup (\widehat X \backslash \bigcap_{i\in I''}K_i) \\ & = \widehat X \backslash K. \end{align*} Überdies ist $K$ kompakt und abgeschlossen in $(X,\tau)$. Das heißt, $U \in \widehat \tau$. Seien nun $U,V \in \widehat \tau$ und \[W = U \cap V.\] Wenn $U,V \in \tau$, so ist $W \in \tau$, da $\tau$ eine Topologie (auf $X$) ist. Angenommen, $U \in \tau$ und $V \notin \tau$. Dann gibt es $L$ kompakt und abgeschlossen in $(X,\tau)$, so dass $V = \widehat X \backslash L$. Es gilt: \[W = U \cap (\widehat X \backslash L) = U \cap (X \backslash L).\] Da $L$ abgeschlossen in $(X,\tau)$ ist, haben wir $X\backslash L \in \tau$. Es folgt $W \in \tau$. Wenn $U \notin \tau$ und $V \in \tau$, so ergibt sich $W \in \tau$ analog (Symmetrie). Angenommen also, $U,V \notin \tau$. Dann existieren $L_1,L_2$ abgeschlossen und kompakt in $(X,\tau)$, so dass \[U = \widehat X \backslash L_1, \qquad V = \widehat X \backslash L_2.\] Mit $L_1$ und $L_2$ ist auch $L_1 \cup L_2$ kompakt und abgeschlossen in $(X,\tau)$. Daher gilt: \[W = (\widehat X \backslash L_1) \cap (\widehat X \backslash L_2) = \widehat X \backslash (L_1 \cup L_2) \in \widehat\tau.\]
Schritt 2. Zu zeigen bleibt die Kompaktheit von $(\widehat X,\widehat\tau)$. Sei dazu $(U_i)_{i\in I}$ eine offene Überdeckung von $(\widehat X,\widehat\tau)$. Dann gibt es $i_0 \in I$, so dass $\infty \in U_{i_0}$. Offensichtlich gilt $U_{i_0} \notin \tau$. Das heißt, es gibt $K$ kompakt und abgeschlossen in $(X,\tau)$, so dass $U_{i_0} = \widehat X \backslash K$. Sei $I' = I \backslash \{i_0\}$ und $(V_i)_{i \in I'}$ die durch \[V_i = U_i \cap X = U_i \backslash \{\infty\}\] erklärte Familie. Dann ist $(V_i)$ eine Familie von Elementen von $\tau$. Zudem überdeckt $(V_i)$ die Menge $K$, denn für $x \in K$ existiert $i \in I$, so dass $x \in U_i$; folglich ist $i \neq i_0$, also $i \in I'$, und überdies gilt $x \in U_i \cap X = V_i$, da $K \subset X$. Da $K$ kompakt in $(X,\tau)$ ist, existiert eine endliche Teilmenge $J' \subset I'$, so dass $(V_i)_{i \in J'}$ die Menge $K$ immer noch überdeckt. Mit $J = J' \cup \{i_0\}$ überdeckt dann $(U_i)_{i\in J}$ die Menge $\widehat X$, denn für $x \in \widehat X$ gilt entweder $x \in U_{i_0}$ oder $x \in \widehat X \backslash U_{i_0} = K$. In jedem Fall gibt es einen Index $i \in J$, so dass $x \in U_i$. $\Box$
Bemerkung.  Ein topologischer Raum der Form $\widehat{(X,\tau)}$ heißt Alexandroff-Kompaktifizierung oder auch Einpunktkompaktifizierung von $(X,\tau)$. Man beachte, dass das in der Aufgabenstellung definierte $\widehat{(X,\tau)}$ von der Wahl von $\infty \notin X$ abhängt. Eine kanonische Wahl von $\infty$ wäre etwa $\infty = X$, denn es gilt $X \notin X$ nach dem Fundierungsaxiom (zusammen mit dem Paarmengenaxiom) der Zermelo-Fraenkel-Mengenlehre (ZF). Auf diese Weise ordnet man einem gegebenen topologischen Raum $X$ eine eindeutig bestimmte Alexandroff-Kompaktifizierung zu.
Zusatzaufgabe. Man beweise: a) $X$ liegt genau dann dicht in $\widehat{(X,\tau)}$, wenn $(X,\tau)$ nicht kompakt ist. b) $\widehat{(X,\tau)}$ ist genau dann Hausdorff, wenn $(X,\tau)$ lokalkompakt und Hausdorff ist.

Aufgabe 3


Wir wissen (aus der Analysis-Vorlesung): Setzt man \[e := (0,\dots,0,1) \in \mathbb R^{n+1},\] so ist \[p \colon S^n \backslash \{e\} \to \mathbb R^n,\] die stereographische Projektion, ist ein Homöomorphismus. Sei \[\widehat p \colon S^n \to \widehat{\mathbb R^n}\] diejenige Erweiterung von $p$, die $e$ auf $\infty$ abbildet. Dann ist $\widehat p$ sicherlich eine Bijektion (da $\infty \notin \mathbb R^n$). Sei $V$ eine offene Teilmenge von $\widehat{\mathbb R^n}$. Wenn $V$ bereits offen in $\mathbb R^n$ ist, so ist \[{\widehat p}^{-1}(V) = p^{-1}(V)\] offen in $S^n$, da $p$ stetig und $S^n \backslash \{e\}$ offen in $S^n$ ist. Wenn $V$ nicht offen in $\mathbb R^n$ ist, so gilt \[V = \widehat{\mathbb R^n} \backslash K\] für eine kompakte, abgeschlossene Teilmenge $K$ von $\mathbb R^n$. Folglich gilt dann \[{\widehat p}^{-1}(V) = S^n \backslash {\widehat p}^{-1}(K) = S^n \backslash p^{-1}(K),\] und diese Menge ist offen in $S^n$, da $p^{-1}(K)$ abgeschlossen in $S^n\backslash \{e\}$ und mithin abgeschlossen in $S^n$ ist. Sei umgekehrt $U$ offen in $S^n$. Wenn $e \notin U$, so ist $U$ offen in $S^n\backslash\{e\}$ und somit \[\widehat p(U) = p(U)\] offen in $\mathbb R^n$ und $\widehat{\mathbb R^n}$. Angenommen, $e \in U$. Dann ist $L := S^n \backslash U$ abgeschlossen und kompakt in $S^n$ und $S^n\backslash\{e\}$. Daher ist $p(L)$ abgeschlossen und kompakt in $\mathbb R^n$, und \[\widehat p(U) = \widehat{\mathbb R^n} \backslash \widehat p(L) = \widehat{\mathbb R^n} \backslash p(L)\] ist offen in $\widehat{\mathbb R^n}$. Folglich ist $\widehat p$ ein Homöomorphismus zwischen $S^n$ und $\widehat{\mathbb R^n}$, und $\widehat{\mathbb R^n}$ ist zu $S^n$ homöomorph. $\Box$

Aufgabe 4


a) Die Abbildung \[\mathrm{Spur} \colon M \to \mathbb R\] ist offensichtlich stetig. Die Menge $\mathbb R \backslash \{0\}$ ist offen in $\mathbb R$. Daher ist \[N = \{A \in M : \mathrm{Spur}(A) \neq 0\} = \mathrm{Spur}^{-1}(\mathbb R \backslash \{0\})\] offen in $M$. $N$ liegt genau dann dicht in $M$, wenn $\overline N = M$. Sei $A \in M$. Wenn $A \in N$, dann auch $A \in \overline N$. Angenommen also, $A \notin N$, das heißt, $\mathrm{Spur}(A) = 0$. Sei $B$ diejenige $n\times n$-Matrix, die durch \[B_{ij} = \begin{cases} 1, & \text{wenn } i=j=1; \\ 0 & \text{sonst}\end{cases}\] für $i,j \in \{1,\dots,n\}$ gegeben ist. Sei $U$ offen in $M$ mit $A \in U$. Dann existiert $\epsilon > 0$, so dass \[B_\epsilon(A) = \{C \in M : ||C-A|| < \epsilon\} \subset U,\] wobei $||-||$ den Rückzug der $n^2$-dimensionalen euklidischen Norm entlang des Isomorphismus $M \to \mathbb R^{n^2}$ bezeichnet. Es gilt \[\mathrm{Spur}(A+\frac\epsilon2B)=\mathrm{Spur}(A) + \frac\epsilon2\mathrm{Spur}(B)=\frac\epsilon2 \neq 0,\] also \[A+\frac\epsilon2B \in N,\] und zudem \[||(A+\frac\epsilon2B)-A|| = \frac\epsilon2||B|| = \frac\epsilon2 < \epsilon,\] also \[A+\frac\epsilon2B \in B_\epsilon(A).\] Somit ist \[U \cap N \neq \emptyset,\] und folglich gilt \[A \in \overline N,\] was zu beweisen war. $\Box$

b) Seien $U$ und $V$ die Urbildmengen von $(0,+\infty)$ und $(-\infty,0)$ respektive bezüglich der Abbildung $\mathrm{Spur}$. Dann sind $U$ und $V$ offen in $M$; es gilt \[U \cup V = N\] und \[U \cap V = \emptyset.\] Zudem sind $U$ und $V$ nichtleer, da \[B \in U, \qquad -B \in V.\] Demzufolge ist $N$ nicht zusammenhängend in $M$. $\Box$

Aufgabe 5


Angenommen, es gibt eine stetige Abbildung $F \colon D^{n+1} \to X$ mit $F \mid S^n = f$. Die Abbildung \[m \colon \mathbb R^{n+1} \times \mathbb R \to \mathbb R^{n+1}, \quad m(x,t) = t \cdot x\] ist offensichtlich stetig (Analysis). Damit sind auch ihre Einschränkung \[m \mid S^n \times I \colon S^n \times I \to D^{n+1}\] sowie die Komposition \[H := F \circ (m \mid S^n\times I) \colon S^n \times I \to D^{n+1} \to X\] stetig. Zudem gilt für alle $x \in S^n$: \[H(x,0) = F(0,\dots,0) =: x_0\] und \[H(x,1) = F(x) = f(x).\] Das heißt, $H$ ist eine Homotopie zwischen der konstanten Abbildung $S^n \to X$ mit Wert $x_0$ und der Abbildung $f \colon S^n \to X$. Folglich ist $f \colon S^n \to X$ nullhomotop.

Nehmen wir umgekehrt $f \colon S^n \to X$ als nullhomotop an. Dann gibt es ein Element $x_0 \in X$, so dass $f$ zur konstanten Abbildung $S^n \to X$ mit Wert $x_0$ homotop ist. Das heißt, es gibt eine Homotopie \[H \colon S^n \times I \to X\] zwischen der genannten konstanten Abbildung und $f$. Wir definieren eine Abbildung \[F \colon D^{n+1} \to X\] durch \[F(x) = \begin{cases} H(x/||x||,||x||), & \text{wenn }x\neq 0; \\ x_0, & \text{wenn }x=0.\end{cases}\] Dann gilt, für alle $x \in S^n$: \[F(x) = H(x,1) = f(x).\] Das heißt, es gilt \[F \mid S^n = f.\] Zudem ist $F$ offensichtlich in allen Punkten $x \in D^{n+1}$, $x \neq 0$ stetig. Sei nun $V$ offen in $X$ mit $F(0) = x_0 \in V$. Wegen der Stetigkeit von $H$ ist dann $H^{-1}(V)$ offen in $S^n \times I$ mit \[S^n \times \{0\} \subset H^{-1}(V).\] Nach unten stehendem Lemma (man beachte: $S^n$ ist kompakt) gibt es daher eine offene Umgebung $W$ von $0$ in $I$, so dass \[S^n \times W \subset H^{-1}(V).\] Es gibt also eine reelle Zahl $\epsilon > 0$, so dass \[S^n \times [0,\epsilon) \subset H^{-1}(V).\] Damit gilt \[B_\epsilon(0) \subset F^{-1}(V),\] wie man unmittelbar aus der Definition von $F$ herausliest. Folglich ist $F^{-1}(V)$ eine Umgebung von $0$ in $D^{n+1}$. Die Abbildung $F \colon D^{n+1} \to X$ ist mithin stetig in $0$, also mithin stetig (schlechthin). $\Box$
Lemma. Seien $K$ und $Y$ topologische Räume, $K$ kompakt, $y_0 \in Y$ und $U$ offen in $K \times Y$, so dass \begin{equation}K \times \{y_0\} \subset U. \label{e_fibersubset}\end{equation} Dann gibt es eine offene Umgebung $W$ von $y_0$ in $Y$, so dass \[K \times W \subset U.\]
Beweis. Wir betrachten das Mengensystem aller offenen Teilmengen $A$ von $K$, so dass eine offene Menge $B$ in $Y$ existiert mit $y_0 \in B$ und $A \times B \subset U$. Aus \eqref{e_fibersubset} folgt, dass dieses Mengensystem $K$ überdeckt. Da $K$ kompakt ist, existieren also eine natürliche Zahl $n\neq 0$ sowie $n$-Tupel $(A_1,\dots,A_n)$ und $(B_1,\dots,B_n)$ offener Mengen in $K$ und offener Umgebungen von $y_0$ in $Y$ respektive, so dass \[K \subset \bigcup_{i=1}^n A_i\] und, für alle $i \in \{1,\dots,n\}$: \[A_i \times B_i \subset U.\] Damit existiert \[W = \bigcap_{i=1}^n B_i,\] und es gilt \[K \times W \subset U;\] zudem ist $W$ offen in $Y$ mit $y_0 \in W$. $\Box$

Montag, 13. Mai 2013

Übungen zur Geometrie: Lösungen zu Blatt 3

Aufgabe 1


(a). Angenommen, $X/R$ ist Hausdorff. Dann gilt nach Blatt 2, Aufgabe 3, dass \[\Delta := \{(a,a):a \in X/R\}\] abgeschlossen in $X/R \times X/R$ ist. Da die Abbildung \[\pi \colon X \to X/R\] stetig ist, ist auch \[\pi \times \pi \colon X \times X \to (X/R) \times (X/R), \quad (\pi\times\pi)(x,y) = (\pi(x),\pi(y))\] stetig. Damit ist die Menge \[(\pi\times\pi)^{-1}(\Delta)\] abgeschlossen in $X \times X$ (vgl. Blatt 1, Aufgabe 3). Sei $(x,y)$ ein Element von $(\pi\times\pi)^{-1}(\Delta)$. Dann gilt $(\pi(x),\pi(y)) \in \Delta$, also $\pi(x) = \pi(y)$, also $x \sim y$, also $(x,y) \in \widetilde R$. Ist umgekehrt $(x,y) \in \widetilde R$, so gilt $x \sim y$, also $\pi(x) = \pi(y)$, also $(\pi(x),\pi(y)) \in \Delta$, also $(x,y) \in (\pi\times\pi)^{-1}(\Delta)$. Damit haben wir \begin{equation}(\pi\times\pi)^{-1}(\Delta) = \widetilde R. \label{e_delta}\end{equation} Folglich ist $\widetilde R$ abgeschlossen in $X\times X$. $\Box$

(b). Wir nehmen an, dass $\widetilde R$ abgeschlossen in $X\times X$ und $\pi \colon X \to X/R$ offen ist. Aus \eqref{e_delta} folgt unmittelbar, dass \[(\pi\times\pi)(\widetilde R) \subset \Delta;\] da $\pi$ surjektiv auf $X/R$ und mithin $\pi\times\pi$ surjektiv auf $X/R \times X/R$ ist, ergibt sich zudem die umgekehrte Inklusion: für $z \in \Delta$ existiert $w \in X\times X$, so dass $(\pi\times\pi)(w)=z$; nach \eqref{e_delta} ist $w \in \widetilde R$, also $z \in (\pi\times\pi)(\widetilde R)$. Nach Definition der Produkttopologie ist mit $\pi \colon X\to X/R$ auch die Abbildung \[\pi\times\pi \colon X\times X \to X/R \times X/R\] offen. Somit ist \[(\pi\times \pi)(X\times X \backslash \widetilde R)\] offen in $X/R \times X/R$ und folglich \begin{equation}(X/R \times X/R) \backslash ((\pi\times \pi)(X\times X \backslash \widetilde R)) \label{e_set}\end{equation} abgeschlossen in $X/R \times X/R$. Sei $z \in \eqref{e_set}$. Dann existiert aufgrund der Surjektivität von $\pi\times\pi$ ein Element $w \in X\times X$, so dass $(\pi\times\pi)(w)=z$. Da $z \notin (\pi\times\pi)(X\times X \backslash \widetilde R)$, ist $w \notin X \times X \backslash \widetilde R$. Damit ist aber $w \in \widetilde R$, also $z \in (\pi\times\pi)(\widetilde R) = \Delta$. Sei umgekehrt $z \in \Delta$. Dann ist $z \in X/R \times X/R$ und wegen \eqref{e_delta} ist $z \notin (\pi\times\pi)(X\times X \backslash \widetilde R)$. Das heißt, $z \in \eqref{e_set}$. Wir haben also \[(X/R \times X/R) \backslash ((\pi\times \pi)(X\times X \backslash \widetilde R)) = \Delta.\] Insbesondere ist $\Delta$ abgeschlossen in $X/R \times X/R$. Nach Blatt 2, Aufgabe 3 ist $X/R$ Hausdorff. $\Box$

Aufgabe 2


Seien $p,q \in \mathbb P_n(\mathbb C)$, so dass $p \neq q$. Wir bezeichnen mit \[S := \{z \in \mathbb C^{n+1}:|z| = 1\}\] die euklidische Sphäre in $\mathbb C^{n+1}$ (identifiziert man $\mathbb C^{n+1}$ mit $\mathbb R^{2n+2}$, so entspricht $S$ der $S^{2n+1}$, was hier jedoch nichts zur Sache tut ...) und setzen \[ p' := p \cap S, \qquad q' := q \cap S, \qquad d := \mathrm{dist}(p',q')\] sowie \begin{align*} U' & := \{x \in S:\mathrm{dist}(\{x\},p') < \frac13d\}, \\ V' & := \{y \in S:\mathrm{dist}(\{y\},q') < \frac13d\} \end{align*} und \[U := \pi(U'), \qquad V := \pi(V'),\] wobei \[\pi \colon \mathbb C^{n+1}\backslash\{0\} \to \mathbb P_n(\mathbb C)\] für die Quotientenabbildung und \[\mathrm{dist}(A,B) := \inf\{|z-w|:z\in A,w\in B\}\] für den euklidischen Abstand zweier Teilmengen $A,B \subset \mathbb C^{n+1}$ steht. Wegen $p \neq q$ haben wir $p \cap q = \emptyset$, also auch $p' \cap q' = \emptyset$. Da $S$ kompakt in $\mathbb C^{n+1}$ und $p' = p \cap S$ (relativ) abgeschlossen in $S$ ist, ist $p'$ kompakt in $\mathbb C^{n+1}$ (vgl. Blatt 2, Aufgabe 2). Ebenso ist $q'$ kompakt in $\mathbb C^{n+1}$. Daher wird der Abstand $d$ von $p'$ und $q'$ angenommen – das Infimum ist ein Minimum –, und es gilt $d > 0$. Mithin haben wir $p' \subset U'$, also $p \in U$, und analog $q \in V$. Definieren wir \[f \colon \mathbb C^{n+1}\backslash \{0\} \to S, \quad f(z) = \frac{z}{|z|},\] so zeigt eine kurze Überlegung, dass \[\pi^{-1}(U) = \pi^{-1}(\pi(U')) = f^{-1}(U').\] [Sei $z \in \pi^{-1}(\pi(U'))$. Dann existiert $x\in U'$, so dass $\pi(z) = \pi(x)$, also $z \sim x$, also $z = \lambda x$ für ein $\lambda \in \mathbb C$, $\lambda \neq 0$. Es folgt $|\lambda| = |z|$. Also ist $f(z) = \mu x$ mit $\mu \in \mathbb C$, $|\mu| = 1$. Für $x' \in p'$ gilt also \[|f(z)-x'| = |\mu x-x'| = |x-\mu^{-1}x'|<\frac13d,\] denn $\mu^{-1}x' \in p'$. Das heißt, $f(z) \in U'$, also $z \in f^{-1}(U')$. Ist umgekehrt $z \in f^{-1}(U')$. Dann gilt $z/|z| \in U'$ und $z/|z| \sim z$, also $\pi(z/|z|) = \pi(z)$, also $z \in \pi^{-1}(\pi(U'))$.] Somit ist $U$ offen in $\mathbb P_n(\mathbb C)$, da $U'$ (relativ) offen in $S$ und $f$ stetig ist. Analog ergibt sich, dass $V$ offen in $\mathbb P_n(\mathbb C)$ ist. Angenommen, es wäre $U \cap V \neq \emptyset$. Dann gäbe es $x \in U'$ und $y \in V'$, so dass $\pi(x) = \pi(y)$, also $x \sim y$, also $y = \lambda x$ für eine komplexe Zahl $\lambda$. Es folgt $|\lambda| = 1$, also $\lambda x \in U'$ (siehe Argument für $f(z)$ oben), also $y \in U' \cap V'$. Daher existieren $x' \in p'$ und $y' \in q'$, so dass $|y-x'|<\frac13d$ und $|y-y'| < \frac13d$. Wir hätten also \[d = \mathrm{dist}(p',q') \leq |x'-y'| \leq |x'-y|+|y-y'| < \frac23 d\] – ein Widerspruch. Damit gilt $U \cap V = \emptyset$. Und $\mathbb P_n(\mathbb C)$ ist ein Hausdorff-Raum. $\Box$

Aufgabe 3


Die Behauptung folgt aus Blatt 2, Aufgabe 2, wenn man dort $K = X$ wählt. $\Box$

Aufgabe 4


(a). Die angegebene Aussage ist offensichtlich falsch. Sei $X = (\emptyset,\{\emptyset\})$ der leere topologische Raum, $Y = (\{0,1\},\mathcal P(\{0,1\}))$ ein zweipunktiger Raum mit diskreter Topologie und $f \colon X \to Y$ die leere Abbildung (der Graph der leeren Abbildung ist schlichtweg die leere Menge $\emptyset \subset X \times Y$). Dann ist $f$ stetig (trivialerweise), $X$ ist wegzusammenhängend (trivialerweise), $Y$ jedoch ist nicht wegzusammenhängend, denn jede stetige Abbildung $\gamma \colon I \to Y$ ist konstant (Zusammenhang von $I$); insbesondere gibt es keinen Weg $\gamma$ in $Y$, so dass $\gamma(0)=0$ und $\gamma(1)=1$. Noch verrückter: Aus der angegebenen Aussage kann man folgern, dass jeder(!) topologische Raum wegzusammenhängend ist – absurd. Gemeint war wohl der
Satz. Ist $f\colon X\to Y$ eine stetige, surjektive Abbildung zwischen topologischen Räumen mit wegzusammenhängendem $X$, so ist $Y$ ebenfalls wegzusammenhängend.
Beweis. Seien $c,d \in Y$. Dann existieren, aufgrund der Surjektivität von $f\colon X\to Y$, Elemente $a,b \in X$, so dass $f(a)=c$ und $f(b)=d$. Da $X$ wegzusammenhängend ist, existiert ein Weg $\gamma$ in $X$, so dass $\gamma(0)=a$ und $\gamma(1)=b$. Damit existiert die Komposition $f \circ \gamma$, ein Weg in $Y$ mit $(f\circ \gamma)(0)=f(a)=c$ und $(f\circ \gamma)(1)=f(b)=d$. $\Box$

(b). Wir nehmen an, dass $A$ in $X$ zusammenhängend ist. Seien $U,V$ nichtleere, offene Mengen in $B$ (bezüglich der von $X$ induzierten Topologie). Dann existiert ein Element $x \in U$. Da $U$ offen in $B$ ist, existiert eine in $X$ offene Menge $U'$, so dass $U = U' \cap B$. Da $B \subset \overline A$, gilt $x \in U' \cap \overline A$. Hieraus folgt, dass $U' \cap A \neq \emptyset$, andernfalls wäre $X \backslash U'$ abgeschlossen in $X$ mit $A \subset X \backslash U'$, also $\overline A \subset X \backslash U'$, also $U' \cap \overline A = \emptyset$. Da \[U \cap A = (U' \cap B) \cap A = U' \cap A,\] folgt, dass $U \cap A$ nichtleer ist. Analog ergibt sich, dass $V \cap A$ nichtleer ist. Angenommen, es gelte $U \cup V = B$. Dann gilt \[(U \cap A) \cup (V \cap A) = (U \cup V) \cap A = B \cap A = A.\] Da $A$ zusammenhängend in $X$ ist, folgt, dass \[(U \cap A) \cap (V \cap A) \neq \emptyset.\] Wegen \[(U \cap A) \cap (V \cap A) \subset U \cap V\] folgt weiter, dass $U \cap V \neq \emptyset$. Dies zeigt, dass $B$ in $X$ zusammenhängend ist.

Aufgabe 5


Wir definieren eine Abbildung $f \colon \mathbb R^n \to (S^1)^n$ durch die Vorschrift \[f(x_1,\dots,x_n) = (g(x_1),\dots,g(x_n)),\] wobei $g \colon \mathbb R \to S^1$ die Funktion $g(x) = e^{2\pi xi}$ bezeichnet – schreibt man $\mathbb R^n$ und $(S^1)^n$ als $n$-fache kartesische Selbstprodukte $\mathbb R\times\dots\times\mathbb R$ und $S^1\times\dots\times S^1$, so entspricht $f$ dem $n$-fachen Selbstprodukt von $g \colon \mathbb R \to S^1$. Da $g$ stetig ist, ist (nach Definition der Produkttopologie) auch $f$ stetig. Die Abbildung $f$ ist zudem invariant unter der angegebenen Gruppenwirkung, das heißt, für alle $m \in \mathbb Z^n$ und alle $m \in \mathbb R^n$ haben wir \[f(m \cdot x) = f(x);\] dies folgt unmittelbar aus der Funktionalgleichung der Exponentialfunktion und der Tatsache, dass $e^{2\pi i} = 1$. Somit faktorisiert $f$ durch die Quotientenabbildung $q \colon \mathbb R^n \to \mathbb R^n/\mathbb Z^n$, mit anderen Worten: es existiert eine (und tatsächlich auch nur eine) Abbildung \[\bar f \colon \mathbb R^n/\mathbb Z^n \to (S^1)^n,\] so dass \[f = \bar f \circ q.\] Nach Definition der Quotiententopologie ist mit $f \colon \mathbb R^n \to (S^1)^n$ auch die Abbildung $\bar f \colon \mathbb R^n/\mathbb Z^n \to (S^1)^n$ stetig. Da $g$ die Menge der reellen Zahlen surjektiv auf $S^1$ abbildet, bildet $f$, und folglich auch $\bar f$, surjektiv auf $(S^1)^n$ ab. Die Abbildung $\bar f$ ist außerdem injektiv, da für reelle Zahlen $x$ und $y$ die Beziehung $e^{2\pi x i} = e^{2\pi yi}$ nur dann gilt, wenn $y = m+x$ für eine ganze Zahl $m$ gilt. Damit ist \[\bar f \colon \mathbb R^n/\mathbb Z^n \to (S^1)^n\] eine Bijektion. Die Abbildung $g \colon \mathbb R \to S^1$ ist offen, da sie durch die verschiedenen Zweige des komplexen Logarithmus (und Nachschalten der Skalierung $(2\pi i)^{-1}$) lokal stetig umgekehrt wird. Demzufolge sind auch die Produktabbildung $f$ sowie deren Faktorisierung $\bar f$ offen. $\bar f$ ist also ein Homöomorphismus zwischen $\mathbb R^n/\mathbb Z^n$ und $(S^1)^n$. Mithin ist $\mathbb R^n/\mathbb Z^n$ homöomorph zu $(S^1)^n$. $\Box$

Montag, 6. Mai 2013

Übungen zur Geometrie: Lösungen zu Blatt 2

Aufgabe 1


Bemerkung. In der Aufgabenstellung wird implizit angenommen, dass $a$, $b$ und $c$ paarweise verschieden sind.

(a). Zu zeigen ist, dass es sich bei $\tau$ um eine Topologie auf $X$ handelt. Dass $\tau \subset \mathcal P(X)$, $\emptyset \in \tau$ und $X \in \tau$, ist offensichtlich. Sei $(U_i)_{i \in I}$ eine Familie von Elementen von $\tau$ und \[U = \bigcup_{i\in I}U_i.\] Sei \[\sigma := \{U_i:i\in I\} \backslash \{\emptyset\}.\] Dann gilt offenbar \[U = \bigcup \sigma.\] Zudem ist \[\sigma \subset \tau \backslash \{\emptyset\} = \{\{a,b\},\{c\},\{a,b,c\}\}.\] Wenn $\{a,b,c\} \in \sigma$, so gilt $U = \bigcup \sigma = \{a,b,c\} \in \tau$. Wenn $\{a,b,c\} \notin \sigma$, so gilt eine der folgenden Aussagen:
  1. $\sigma = \emptyset$ und $U = \emptyset \in \tau$.
  2. $\sigma = \{\{a,b\}\}$ und $U = \{a,b\} \in \tau$.
  3. $\sigma = \{\{c\}\}$ und $U = \{c\} \in \tau$.
  4. $\sigma = \{\{a,b\},\{c\}\}$ und $U = \{a,b,c\} \in \tau$.
In jedem Fall haben wir $U \in \tau$. Sind $V,W \in \tau$, so ist $V \cap W \in \tau$, wie man leicht vermittels Fallunterscheidung nachweist: Ist $V = \emptyset$ oder $W = \emptyset$, so ist $V \cap W = \emptyset \in \tau$. Ist $V = X$, so ist $V \cap W = W \in \tau$. Ist $W = X$, so ist $V \cap W = V \in \tau$. Liegt keiner dieser Fälle vor, so gilt eine der folgenden Aussagen:
  1. $V = W = \{a,b\}$ und folglich $V \cap W = \{a,b\} \in \tau$.
  2. $V = W = \{c\}$ und folglich $V \cap W = \{c\} \in \tau$.
  3. $V = \{a,b\}$ und $W = \{c\}$ und folglich $V \cap W = \emptyset \in \tau$.
(b). Behauptung. Eine Folge $(x_n)_{n\in\mathbb N}$ in $X$ ist genau dann in $(X,\tau)$ konvergent, wenn eine der folgenden beiden Aussagen gilt:
  1. Es existiert $N \in \mathbb N$, so dass $x_n \in \{a,b\}$ für alle $n \in \mathbb N$ mit $N<n$.
  2. Es existiert $N \in \mathbb N$, so dass $x_n = c$ für alle $n \in \mathbb N$ mit $N<n$.
Beweis. Gilt Aussage 1, so ist $(x_n)$ offensichtlich konvergent gegen $a$ (resp. $b$), da $\{a,b\}$ und $X$ die einzigen offenen Mengen in $(X,\tau)$ sind, die $a$ (resp. $b$) enthalten. Gilt Aussage 2, so ist $(x_n)$ offensichtlich konvergent gegen $c$. Ist umgekehrt $(x_n)$ konvergent in $(X,\tau)$, so existiert $l \in X$, so dass $(x_n)$ konvergent gegen $l$ ist. Da $X = \{a,b,c\}$, gilt $l=a$ oder $l=b$ oder $l=c$. Wenn $l=a$, so gilt Aussage 1, denn $\{a,b\}$ ist eine offene Menge in $(X,\tau)$, die $a$ enthält. Wenn $l=b$, so gilt Aussage 1, denn $\{a,b\}$ ist eine offene Menge in $(X,\tau)$, die $b$ enthält. Wenn $l=c$, so gilt Aussage 2, denn $\{c\}$ ist eine offene Menge in $(X,\tau)$, die $c$ enthält, und (für alle $n\in \mathbb N$) bedingt $x_n \in \{c\}$, dass $x_n = c$. $\Box$

(c). Wir definieren eine Folge $(x_n)$ durch: \[x_n = \begin{cases}a, & \text{wenn $n$ gerade;} \\ c, & \text{wenn $n$ ungerade.}\end{cases}\] Dann ist $(x_n)$ eine Folge in $X$, die im topologischen Raum $(X,\tau)$ divergent ist, da sie keiner der beiden Bedingungen aus Teil (b) genügt.

(d). Es sei $(x_n)$ die konstante Folge mit Wert $a$. Dann ist $(x_n)$ in $(X,\tau)$ sowohl konvergent gegen $a$ als auch konvergent gegen $b$ (vgl. Teil (b)). Tatsächlich gilt, wie man sich leicht überlegt: Jede in $(X,\tau)$ gegen $a$ (resp. $b$) konvergente Folge konvergiert auch gegen $b$ (resp. $a$).

Aufgabe 2


Beweis. Sei $(U_i)_{i\in I}$ eine offene Überdeckung von $A$ in $(X,\tau)$. Wir wissen, es existiert $e$, so dass $e \notin I$ (zum Beispiel $e = I$ ...). Damit existiert auch $I' = I \cup \{e\}$ sowie die Familie $(V_i)_{i\in I'}$, die durch $V_i = U_i$ für $i \in I$ und $V_e = X \backslash A$ gegeben ist. Da $A$ abgeschlossen in $(X,\tau)$ ist, ist $X \backslash A$ offen in $(X,\tau)$ und $(V_i)$ ist eine offene Überdeckung von $K$ in $(X,\tau)$. Da $K$ in $(X,\tau)$ kompakt ist, existiert eine endliche Teilmenge $J' \subset I'$, so dass $(V_i)_{i\in J'}$ die Menge $K$ immer noch überdeckt. Mit $J = J' \backslash \{e\}$ ist $J$ eine endliche Teilmenge von $I$, so dass $(U_i)_{i\in J}$ die Menge $A$ überdeckt. $\Box$

Aufgabe 3


Es gilt:
\begin{align*}& X \text{ Hausdorff} \\ & \Leftrightarrow (\forall x,y \in X)(x\neq y \rightarrow (\exists U,V\in\tau) x\in U, y\in V, U\cap V = \emptyset) \\ & \Leftrightarrow (\forall (x,y) \in X\times X \backslash \Delta)(\exists U,V\in\tau) x\in U, y\in V, U\cap V = \emptyset \\ & \Leftrightarrow (\forall (x,y) \in X\times X \backslash \Delta)(\exists U,V\in\tau) (x,y) \in U\times V \subset X\times X\backslash \Delta \\ & \Leftrightarrow X\times X\backslash \Delta \text{ offen in }X\times X \\ & \Leftrightarrow \Delta \text{ abgeschlossen in }X\times X.\end{align*}
Man beachte bei dieser Argumentation die Definition der Produkttopologie (auf $X \times X$) sowie die Tatsache, dass \[z \in U \cap V \quad \leftrightarrow \quad (z,z) \in U\times V.\]

Aufgabe 4

Lemma. Seien $X,Y$ topologische Räume und $f \colon X\to Y$ eine stetige Abbildung. Dann ist die Abbildung $\tilde f$, definiert durch $\tilde f(x) = (x,f(x))$ für alle $x \in X$, ein Homöomorphismus zwischen $X$ und dem Graph $G \subset X\times Y$ von $f$, der mit der induzierten Topologie ausgestattet ist.
 Beweis. Da $\mathrm{id}_X \colon X \to X$ und $f \colon X \to Y$ stetig sind, ist \[\tilde f = (\mathrm{id}_X,f) \colon X \to X \times Y\] stetig nach der universellen Eigenschaft des Produkts zweier topologischer Räume. Offenkundig ist $\tilde f$ eine Abbildung $X \to G$. Da $G$ mit der Teilraumtopologie von $X\times Y$ ausgestattet ist, ist die Abbildung $\tilde f\colon X \to G$ stetig. Wie man sich leicht überlegt, ist $\tilde f\colon X \to G$ bijektiv mit der Umkehrabbildung $p_0|G \colon G \to X$, wobei \[p_0 \colon X\times Y \to X\] die Projektionsabbildung auf den ersten Faktor bezeichnet. Erneut nach Definition der Teilraumtopolgie ist die Einschränkung $p_0|G \colon G \to X$ stetig und somit $\tilde f$ ein Homöomorphismus zwischen $X$ und $G$. $\Box$

Nach dem Lemma (für $X=M$ und $Y=\mathbb R$) ist klar, dass $\tilde f \colon M \to G$ ein Homöomorphismus ist, wobei $G$ die Teilraumtopologie des Produkts $M \times \mathbb R$ trägt. Der Raum $\Gamma$ stimmt mengentheoretisch mit $G$ überein, ist jedoch mit der Unterraumtopologie des euklidischen $\mathbb R^{n+1}$ ausgestattet. Zwei Überlegungen zeigen, dass $G$ und $\Gamma$ auch als topologische Räume übereinstimmen:
  1. $G$ trägt die Teilraumtopologie von $\mathbb R^n \times \mathbb R$, wobei letzterer Raum mit der Produkttopologie der euklidischen Topologien auf $\mathbb R^n$ und $\mathbb R$ versehen ist.
  2. Die Produkttopologie auf $\mathbb R^n \times \mathbb R$ ist gleich der euklidischen Topologie auf $\mathbb R^{n+1}$ (sofern man die unterliegenden Mengen auf kanonische Weise miteinander identifiziert).

Aufgabe 5


(a). Da $A \cap B \subset A$, folgt $(A \cap B)^\circ \subset A^\circ$. Analog folgt aus $A \cap B \subset B$, dass $(A \cap B)^\circ \subset B^\circ$. Damit gilt $(A \cap B)^\circ \subset A^\circ \cap B^\circ$. Sei umgekehrt $x \in A^\circ \cap B^\circ$. Dann ist $x \in A^\circ$. Folglich existiert eine (im topologischen Raum $X$) offene Menge $U$, so dass $x \in U \subset A$. Analog existiert eine offene Menge $V$, so dass $x \in V \subset B$. Demzufolge existiert $W = U \cap V$, eine offene Menge, so dass $x \in W \subset A \cap B$. Mithin ist $x \in (A \cap B)^\circ$.

(b). Augenscheinlich gilt: \[A^\circ \cup B^\circ \subset (A \cup B)^\circ.\] Die umgekehrte Inklusion gilt im Allgemeinen nicht, wie das folgende Beispiel zeigt. Es sei $X=\mathbb R$ (euklidisch topologisiert), $A = \mathbb Q$, $B = \mathbb R \backslash \mathbb Q$. Dann gilt $A^\circ = B^\circ = \emptyset$, also auch $A^\circ \cup B^\circ = \emptyset$, jedoch: \[(A \cup B)^\circ = {\mathbb R}^\circ = \mathbb R.\]

(c). Offenbar gilt: \[\overline{A \cap B} \subset \overline A \cap \overline B.\] Die umgekehrte Inklusion gilt wiederum (im Allgemeinen) nicht, wie das Beispiel aus (b) zeigt – dort ist $\overline{A \cap B} = \emptyset$, jedoch $\overline A \cap \overline B = \mathbb R$.

(d). Diese Aussage ist äquivalent zu (a). Genauer gesagt gilt: \begin{align*}& \overline{A\cup B} = \overline A \cup \overline B \\ \Leftrightarrow & X \backslash \overline{A\cup B} = X \backslash (\overline A \cup \overline B) \\ \Leftrightarrow & (X \backslash (A \cup B))^\circ = (X \backslash \overline A) \cap (X \backslash \overline B) \\ \Leftrightarrow & (X \backslash A \cap X \backslash B)^\circ = (X \backslash A)^\circ \cap (X \backslash B)^\circ,\end{align*} wobei wir mehrfach die Beziehung \[(X \backslash C)^\circ = X \backslash \overline C\] verwendet haben.